Administración     

Olimpiadas de Matemáticas
Página de preparación y problemas

Selector
La base de datos contiene 1154 problemas y 775 soluciones.
OME Local
OME Nacional
OIM
OME Andalucía
Retos UJA
Problema 651
La última cifra de $2009^{2011}$ es un nueve pero, ¿cuántos ceros preceden a ese nueve?
pistasolución 1solución 2info
Pista. Desarrolla $(2010-1)^{2011}$ usando el binomio de Newton. Otra alternativa es usar directamente el teorema de Euler-Fermat.
Solución. Podemos desarrollar la potencia usando el binomio de Newton si escribimos la base como $2010-1$. Además, trabajaremos módulo $1000$ porque solo nos van a interesar las tres últimas cifras. Tenemos que \[2009^{2011}\equiv 9^{2011}\equiv(10-1)^{2011}\equiv-1+\binom{2011}{1}\cdot 2010-\binom{2011}{2}\cdot 10^2.\] No tenemos que poner más términos del binomio porque a partir del siguiente nos quedan múltiplos de $1000$. De esta forma, podemos seguir desarrollando módulo $1000$ los números combinatorios para escribir finalmente: \[2009^{2011}\equiv -1+11\cdot 10+55\cdot 10^2\equiv-391\equiv 609.\] Por lo tanto, las tres últimas cifras son $609$ y concluimos que solo hay un cero que precede al nueve.

Nota. Si necesitáramos más cifras sólo habría que trabajar módulo una potencia de \(10\) más grande y añadir más términos al desarrollo del binomio. Esto quiere decir que no es relevante en el problema que solo haya que calcular tres dígitos.

Solución. Vamos a trabajar módulo $1000$ ya que solo nos van a interesar las tres últimas cifras. Vamos a usar el teorema de Euler-Fermat que nos dice que $a^{\varphi(n)}\equiv 1\ (\text{mod }n)$ si $a$ y $n$ son primos relativos y $\varphi(n)$ es la función de Euler. En nuestro caso, pondremos $a=9$ y $n=1000$, usando que $\varphi(1000)=\varphi(2^3\cdot 5^3)=2^2(2-1)\cdot 5^2(5-1)=400$. Por lo tanto, \[2009^{2011}\equiv 9^{2011}=(9^{400})^5\cdot 9^{11}\equiv 9^{11}\ (\text{mod }1000).\] Ahora bien, este último resto lo podemos calcular, observando que \[9^2\equiv 81\ (\text{mod }1000),\quad 9^3\equiv 729\ (\text{mod }1000)\] para luego calcular rápidamente \[9^{11}\equiv 9^3\cdot 9^3\cdot 9^3\cdot 9^2\equiv 729\cdot 729\cdot 729\cdot 81\equiv 609\ (\text{mod }1000).\] Aunque pueda parecer una cuenta larga, sólo nos tenemos que quedar con las últimas tres cifras en cada paso. Deducimos así que las tres últimas cifras de $2009^{2011}$ son $609$, con lo que la respuesta a la pregunta del enunciado es uno.
Si crees que el enunciado contiene un error o imprecisión o bien crees que la información sobre la procedencia del problema es incorrecta, puedes notificarlo usando los siguientes botones:
Informar de error en enunciado Informar de procedencia del problema
Problema 650
Consideremos la sucesión de enteros positivos $\{x_n\}$ definida por $x_1=2$ y $x_{n+1}=2x_n^3+x_n$ para todo $n\geq 1$. Hallar la mayor potencia de $5$ que divide a $x_{2014}^2+1$.
pistasolución 1info
Pista. Demuestra por inducción que $x_n$ es múltiplo de $5^n$ pero no de $5^{n+1}$.
Solución. Se pueden calcular algunos términos, pero rápidamente el resultado se dispara ya que la sucesión crece exponencialmente:\[x_1^2+1=5,\quad x_2^2+1=325,\quad x_3^2+1=136469125,\ldots\] Vamos a probar por inducción sobre $n$ que $x_n^2+1$ es múltiplo de $5^n$ pero no de $5^{n+1}$. Si probamos esto, tendremos que la solución al problema es $5^{2024}$. Está claro que el caso $n=1$ es cierto ya que $x_1^2+1=5$ es múltiplo de $5$ pero no de $25$. También es cierto si $n=2$ ya que $x_2^2+1=325$ es múltiplo de $25$ pero no de $125$. Supongamos que la propiedad es cierta para $n\geq 2$, lo que nos permite escribir $x_n^2+1=5^ny_n$, siendo $y_n$ no múltiplo de $5$. Entonces, para $x_{n+1}$ podemos desarrollar y simplificar \begin{align*}x_{n+1}^2+1&=(2x_n^3+x_n)^2+1=(4x_n^4+4x_n^2+1)x_n^2+1\\\\&=(4(5^ny_n-1)^2+4(5^ny_n-1)+1)(5^ny_n-1)+1\\\\&=4\cdot 5^{3n}y_n^3-8\cdot 5^{2n}y_n^2+5^{n+1}y_n\\\\&=5^{n+1}(4\cdot 5^{2n-1}y_n^3-8\cdot 5^{n-1}y_n^2+y_n).\end{align*}Este número es múltiplo de $5^{n+1}$ pero no de $5^{n+2}$ ya que el factor $4\cdot 5^{2n-1}y_n^3-8\cdot 5^{n-1}y_n^2+y_n$ es congruente con $y_n$ módulo $5$. Aquí estamos usando que $n\geq 2$ para asegurar que $5^{n-1}$ es múltiplo de $5$, es decir, en la inducción hemos tenido que comprobar dos casos iniciales.
Si crees que el enunciado contiene un error o imprecisión o bien crees que la información sobre la procedencia del problema es incorrecta, puedes notificarlo usando los siguientes botones:
Informar de error en enunciado Informar de procedencia del problema
Problema 648
Un número natural $n\lt 1000$ se dice $4$-malagueño si tiene la siguiente propiedad:

Para cualquier múltiplo $N$ de $n$ con cuatro cifras, pongamos $N = {abcd}$, se verifica que todas las permutaciones circulares de $N$ ($N' = {bcda}$, $N'' = {cdab}$, $N''' = {dabc}$) también son múltiplos de $n$. Por ejemplo, $11$ es un número $4$-malagueño.

Determina todos los números $4$-malagueños.
Sin pistas
Sin soluciones
info
Si crees que el enunciado contiene un error o imprecisión o bien crees que la información sobre la procedencia del problema es incorrecta, puedes notificarlo usando los siguientes botones:
Informar de error en enunciado Informar de procedencia del problema
Problema 638
Encontrar todas las soluciones de la ecuación \[nm = k(n + m)\] donde $n$ y $m$ son números enteros y $k$ es un número primo mayor o igual que 2.
pistasolución 1info
Pista. Factoriza la ecuación como $(n-k)(n-k)=k^2$.
Solución. Observemos que la ecuación se puede escribir como \[(m-k)(n-k)=k^2.\] Si suponemos que $m\leq n$, como los divisores de $k^2$ son $\pm 1$, $\pm k$ y $\pm k^2$, tendrá que darse alguna de las siguientes posibilidades:
  • $m-k=-k^2$, $n-k=-1$, de donde $m=k-k^2$ e $n=k-1$,
  • $m-k=-k$, $n-k=-k$, de donde $m=n=0$,
  • $m-k=1$, $n-k=k^2$, de donde $m=k+1$ e $n=k^2+k$,
  • $m-k=k$, $n-k=k$, de donde $m=n=2k$.
Esto nos da la siguientes seis posibilidades para el par $(m,n)$: \begin{align*} &(k-k^2,k-1),&&(k-1,k-k^2),&&(0,0),&\\ &(k+1,k^2+k),&&(k^2+k,k+1),&&(2k,2k).& \end{align*}

Nota. Este fue el problema 4 de la fase nacional de la Olimpiada Matemática Española de 1995.

Si crees que el enunciado contiene un error o imprecisión o bien crees que la información sobre la procedencia del problema es incorrecta, puedes notificarlo usando los siguientes botones:
Informar de error en enunciado Informar de procedencia del problema
Problema 637
Sean $k$, $m$ y $n$ enteros positivos tales que $k+m+1$ es un primo estrictamente mayor que $n+1$. Si $C_s$ denota el entero $s(s+1)$, demostrar que el producto \[(C_{m+1} − C_k)(C_{m+2}-C_k)\cdots(C_{m+n}-C_k)\] es divisible por $C_1C_2\cdots C_n$.
pistasolución 1info
Pista. Observa en primer lugar que puedes factorizar $C_{m+i}-C_k=(m+k+i+1)(m-k+i)$ y expresa el producto del enunciado en términos de números combinatorios.
Solución. En primer lugar, vamos a desarrollar los factores que aparecen en ese producto para obtener una expresión más sencilla. Observamos que \begin{align*} C_{m+i}-C_k&=(m+i+1)(m+i)-(k+1)k\\ &=(m+i)^2-k^2+m+i-k\\ &=(m+k+i)(m-k+i)+m-k+i\\ &=(m+k+i+1)(m-k+i). \end{align*} Por lo tanto, podemos agrupar factores para expresar \begin{align*} &\frac{(C_{m+1}-C_k)\cdots(C_{m+n}-C_k)}{C_1C_2\cdots C_n}\\ &\quad=\frac{(m+k+2)(m-k+1)\cdots(m+k+n+1)(m-k+n)}{1\cdot 2\cdot 2\cdot 3\cdots n(n+1)}\\ &\quad=\frac{(m+k+2)\cdots(m+k+n+1)(m-k+1)\cdots(m-k+n)}{n!(n+1)!}\\ &\quad=\frac{1}{m+k+1}\binom{m+k+n+1}{n+1}\binom{m-k+n}{n}. \end{align*} Los números combinatorios son enteros pero falta por ver que alguno de ellos es divisible por el primo $m+k+1$. Claramente lo es el primero de ellos dado que en \[\binom{m+k+n+1}{n+1}=\frac{(m+k+1)(m+k+2)\cdots(m+k+n+1)}{(n+1)!}\] el numerador es múltiplo de $m+k+1$ y el denominador no lo es ya que $n+1\lt m+k+1$ por hipótesis.
Si crees que el enunciado contiene un error o imprecisión o bien crees que la información sobre la procedencia del problema es incorrecta, puedes notificarlo usando los siguientes botones:
Informar de error en enunciado Informar de procedencia del problema
José Miguel Manzano © 2010-2024. Esta página ha sido creada mediante software libre